A line segment with a length of 16 units is reflected across the y-axis, then translated 5 units down on a coordinate plane, resulting in a new segment.
What is the length of this new line segment? ________________ units

Answers

Answer 1

The solution is 16 units

The length of the line segment is 16 units

How does the transformation of a function happen?

The transformation of a function may involve any change.

Usually, these can be shifted horizontally (by transforming inputs) or vertically (by transforming output), stretched (multiplying outputs or inputs), etc.

If the original function is y = f(x), assuming the horizontal axis is the input axis and the vertical is for outputs, then:

Horizontal shift (also called phase shift):

Left shift by c units:  y=f(x+c) (same output, but c units earlier)

Right shift by c units:  y=f(x-c)(same output, but c units late)

Vertical shift:

Up by d units: y = f(x) + d

Down by d units: y = f(x) - d

Stretching:

Vertical stretch by a factor k: y = k × f(x)

Horizontal stretch by a factor k: y = f(x/k)

Given data ,

Let the length of the line segment be = A

Now , the initial length of the line segment = 16 units

The line is reflected across the y-axis

So , when you reflect a point across the y-axis, the y-coordinate remains the same, but the x-coordinate is taken to be the additive inverse. The reflection of point (x, y) across the y-axis is (-x, y)

And , the resulting line is translated 5 units down

But , the line segment does not change in dimension at all

So , the reflection and a translation does not affect the length of the line segment

Hence , the length of the line segment is 16 units

To learn more about transformation of function click :

https://brainly.com/question/26896273

#SPJ1


Related Questions

Which choice is the equation of the line written in slope-intercept form?

HINT: Use the answer from number 1 to help you with this question. The graph is the same graph used in number 1.

A: y=1/2x - 1/2

B: y=1/2x+3/2

C: y=3/2x+1/2

D: y=3/2x−1/2

Answers

Answer:

I think its B

Step-by-step explanation:

Sorry if im wrong

The answer is B. If you look at the equation on the graph, the y-intercept is 1.5 as 3/2=1.5 and the slope to the point (1, 2) is 1/2 because we rise .5 a unit and run 1 unit. This is represented as .5/1, and any number divide by 1 is itself, so the slope is .5 or 1/2.

In which of the options below will the number 10 correctly fill in the blank? Select all that apply. A) 3 : 5 = 6 : ___ B) 2 : ___ = 4 : 16 C) 1 : ___ = 4 : 40 D) 5 : 8 = ___ : 24

Answers

the answer is C
1: 10 = 4: 40

What is equation of line through 1 2 so that segment of line?

Answers

The equation of the line passing through the point so that segment of the line i.e. [tex]y+2x=4[/tex]

The equation of the line be [tex]y=mx+c[/tex]

Now the line intersects at the x-axis  [tex](a,0)[/tex] and the point at which the y-axis intersect [tex](0,b)[/tex]

Plug in the value [tex](a,0)\\[/tex]

[tex]ma+c=0\\a=\frac{-c}{m}[/tex]

and now plug in the value [tex](b,0)[/tex]

[tex]b=c+0\\b=c[/tex]

now [tex](1,2)[/tex] is the midpoint of [tex](a,0)[/tex] and [tex](0,b)\\[/tex]

[tex]2=\frac{0+b}{2\\}\\ b=c\\\frac{c}{2} =2\\c=4[/tex]

and

[tex]1=\frac{a+0}{2}\\1=\frac{-c}{2m} \\m=-2[/tex]

plug in the value of m and c

[tex]y+2x=4[/tex] is the required equation

Learn more about equations from

https://brainly.com/question/29657983

#SPJ4

What is the pair of angle â  JKL and â  Lkm called?

Answers

If the measure of angle JKL is 132°  and the measure of angle LKM is 48° . then the pair of angle JKL and angle LKM called a Linear Pair .

In the question ,

it is given that ,

the measure of angle JKL is = 132°    and

the measure of angle LKM is = 48°  ,

we know that when the sum of two angles is 180° , then the pair is called as Linear Pair .

According to the question ,

angle JKL + angle LKM = 180°

132° + 48° = 180°

180° = 180°

So , the given pair of angles forms a Linear Pair .

Therefore , the angle JKL and angle LKM forms a Linear pair .

The given question is incomplete , the complete question is

If the measure of angle JKL is 132°  and the measure of angle LKM is 48° . What is the pair of angle JKL and angle LKM called ?

Learn more about Linear Pair here

https://brainly.com/question/17525542

#SPJ4

The choice of the probability distribution for a random variable can be guided by: ________

Answers

The choice of the probability distribution for a random variable can be guided by Historical Data

What is Probability ?

Probability is a branch of mathematics that deals with numerical representations of the likelihood of an event occurring or of a proposition being true. The probability of an event is a number between 0 and 1, with 0 approximately denoting impossibility and 1 denoting certainty.

What is Random Variable Distribution ?

In probability theory and statistics, a probability distribution is a mathematical function that calculates the likelihood of a range of potential experiment results. It is a mathematical explanation of a random phenomenon in terms of its sample space and event probability.

The choice of the probability distribution for a random variable can be guided by Historical Data

To know more about Random Variable Distribution

https://brainly.com/question/29069559

#SPJ4

the scores in an exam were normally distributed with unknown mean and a standard deviation of 5 . if 30.85 % of students scored more than 72.5 . what is the value of the mean? ( round to a whole number

Answers

The value of the mean for the scores in an exam which are normally distributed is 70.

Given:

the scores in an exam were normally distributed with unknown mean and a standard deviation of 5 .

if 30.85 % of students scored more than 72.5 . what is the value of the mean = ?

Mean = µ

σ = 5

x = 72.5

The z score corresponding to the area 30.85% in the right tail is :

Zi score = Z 0.3085

             = 0.500

Again : Z₀ = x- µ/σ

           ⇒ x- µ = Z₀σ

           ⇒ µ = x - Z₀σ

                  = 72.5 - 0.500 × 5    (using z table)

                  = 72.5 - 2.5

                  = 70

Hence we get the unknown mean as 70.

Learn more about Mean here:

brainly.com/question/20118982

#SPJ4

there are 123 students in a drama club. 65 of them are girls. what is the ratio of the numbers of boys to the number girls?

Answers

False because 124 is 12 type

Answer:

Step-by-step explanation:

123(students in general) - 65 (girls) = 58

1. The cost y of movie tickets varies directly
with the number of tickets x as shown in the
graph. Write a direct variation equation to
represent this relationship. Then identify the
constant of variation and interpret its
meaning. (Example 1)
Movie Tickets
Cost ($)
56
48
40
32
24
16
8
y
X
0 1 2 3 4 5 6 7 8 9
Number of Movie Tickets

Answers

The direct variation equation to represent the relationship between the number of tickets and the cost of the ticket is y = 8x.

What is graph?

A graph is a structure made up of a collection of things, where some object pairs are conceptually "connected." The items are represented by mathematical abstractions known as vertices, and each pair of connected vertices is referred to as an edge.

Given:

56, 48, 40, 32, 24, 16, 8, 0, y

7, 6, 5, 4, 3, 2, 1, 0, x

Here, x is the number of tickets and y is the cost of the ticket

As you can see that the cost of each ticket is $8,

Then the equation will be,

y = 8x

To know more about Graph:

https://brainly.com/question/10712002

#SPJ1

three people are asked to throw a fair die. what is the probability that they all get the same number

Answers

As three people are asked to throw a fair die. Then, the probability that they all get the same number is 1/36.

Probability:

Probability is a branch of mathematics that quantifies the likelihood of an event occurring or the likelihood of a statement being true. Probability is a number between 0 and 1, with 0 generally indicating impossibility and 1 indicating certainty that an event will occur. A simple example is tossing a fair (unbiased) coin. Since the coin is fair, the two outcomes (heads and tails) are equally likely. The probability of heads is the same as the probability of tails. Since no other outcome is possible, the probability of heads or tails is 1/2 (also written as 0.5 or 50%).

According to the Question:

Theoretical, Probability is based on the probability that something will happen. Probability justification serves as the main basis for theoretical probability.

Given that,

There is a 1 in 6 chance that the dice will match.

The number of throwers is 3.

That way everyone has a chance to get the same number.

And the probability is:

The total outcome is 3 dice = ( 6 × 6 × 6 = 216 ).

Therefore,

probability of getting the same number = 6 × 1/6 × 1/6 × 1/6

                                                                   = 6 × 1/6 × 1 × 1/6 × 6

                                                                   = 6/6 × 1/36

                                                                   = 1 × 1/36

Therefore,

Probability of all three thrown is 1/36.

Learn more about Probability:

https://brainly.com/question/12905909

#SPJ4

Solve the inequality and write the solution in set-builder notation.
3h+5<4h

Answers

The inequality 3h + 5 < 4h in set builder form is  {h : h ∈ R, and h > 5}.

What are inequalities and their types?

Inequality is a relation that compares two numbers or other mathematical expressions in an unequal way.

The symbol a < b indicates that a is smaller than b.

When a > b is used, it indicates that a is bigger than b.

a is less than or equal to b when a notation like a b.

a is bigger or equal value of an is indicated by the notation a b.

Given, An inequality 3h + 5 < 4h.

3h - 4h < - 5.

-h < - 5.

h > 5                   (Make the variable positive by moving it to the other side)

So, The inequality in the set builder form is {h : h ∈ R, and h > 5}

learn more about set builder form here :

https://brainly.com/question/29531272

#SPJ1

Andrea ordered a computer on the Internet.

The computer cost $1,499 plus 7 12% sales tax.

What was the total amount Andrea paid for her computer? Round to the nearest cent.

Proportions bar also please if possible?

Answers

The total amount Andrea paid for her computer is $1611.425.

What was the total amount Andrea paid for her computer?

A percentage is a value or ratio that may be stated as a fraction of 100. If we need to calculate a percentage of a number, we should divide it's entirety and then multiply it by 100

In this situation, Andrea ordered a computer on the Internet and the computer cost $1,499 plus 7 1/2% sales tax.

The amount that will be paid will be:

= Cost of computer + Tax

= $1499 + (7.5% × $1499)

= $1499 + $112.425

= $1611.425

The amount is $1611.425.

Learn more about percentages on:

brainly.com/question/24877689

#SPJ1

3x - 5(x -5) = - 3 + 5x + 7

Answers

Answer:

x=3

Step-by-step explanation:

3x-5x+25= 5x+4

-2x+25=5x+4

+2x.      +2x

25=7x+4

-4.      -4

21=7x

/7. /7

3=x

Hopes this helps please mark brainliest

Answer:

x = 3

Step-by-step explanation:

3x - 5(x -5) = - 3 + 5x + 7

3x - 5x + 25 = 4 + 5x

3x - 5x -5x = 4 - 25

-7x = -21

x = -21 : (-7)

x = 3

How do I write an expression for the measure of the angle ?

Answers

How do you write angle measurements?

Angles are measured in degrees (°) using a protractor. A protractor is a measuring device that is used to calculate or draw angles in terms of degrees. For example, in the image below, we see that using a protractor, the black arrow points to 100°, crossing 90°. Hence, the measure of the angle is 100°.

The idea of an angle

Degrees are the most widely used unit of measurement. A straight angle is 90° since a circle has 360 equal degrees.

degrees

Angles are measured in degrees (°) using a protractor. A protractor is a measuring device that is used to calculate or draw angles in terms of degrees. For example, in the image below, we see that using a protractor, the black arrow points to 100°, crossing 90°. Hence, the measure of the angle is 100°.

To learn more about angle measurements visit:

https://brainly.com/question/15592639

#SPJ1

For the player with 450 times at bat, the actual number of hits the player had is approximately how many fewer than the number of hits predicted by the line of best fit?.

Answers

the actual number of hits the player had is approximately 10 fewer than the number of hits predicted by the line of best fit

The guy with 450 at bat had about 113 hits, according to the scatterplot (the y-coordinate of the point representing this player is approximately halfway between 100 and 125). The best fit line anticipates around 123 hits. As a result, this player's actual number of hits is around ten lower than the number of hits predicted by the line of best fit.

20,30,40 are inaccurate because they each offer a figure significantly more than 10, which is the best guess of how many fewer hits the player with 450 at-bats made than anticipated by the best-fit line of numbers.

Learn more about lines of best fit here: https://brainly.com/question/17261411

#SPJ4

a survey of people on pizza preferences indicated that 55 percent preferred pepperoni only, 30 percent preferred mushroom only, and 15 percent preferred something other than pepperoni and mushroom. suppose one person who was surveyed will be selected at random. let p represent the event that the selected person preferred pepperoni, and let m represent the event that the selected person preferred mushroom. are p and m mutually exclusive events for the people in this survey?

Answers

Therefore option C) The probability of P and M together is equal to 0, hence the answer is yes.

What does the process of probability mean?

In probability theory, the likelihood that an event will happen or that a statement is true is expressed numerically. A number between 0 and 1, with 1 often denoting certainty and 0 typically denoting impossibility, is used to describe the likelihood of an event.

Here,

As a result, P stands for the possibility that the chosen candidate favored pepperoni exclusively.

And M stands for the situation where the chosen person preferred just mushrooms.

Therefore, P and M are two occurrences that are mutually exclusive since two events are mutually exclusive if they do not occur simultaneously.

P and M cannot occur simultaneously in this case.

As a result, there is no chance that P and M will happen simultaneously, or at all.

To know more about probability , visit

brainly.com/question/11234923

#SPJ4

Which one of the following compound inequalities has no solution?
O 3x +1 < 5x + 7 and -2x + 5
-5x - 10
O 3x - 7≤ 5x +5 and - 4x + 8 < 5x - 1
-2x + 9 4x + 3 and 8x9 <- 7x - 3
O-2(x-9) < 3(x+4) and - 4(x - 1) > - 5(x - 2)

Answers

Inequality -2x + 9 4x + 3 and 8x9 <- 7x - 3 has no solution.

Define inequality.

In mathematics, inequalities specify the connection between two non-equal numbers. Equal does not imply inequality. Typically, we use the "not equal sign ( ≠ )" to indicate that two values are not equal. But several inequalities are utilized to compare the numbers, whether it is less than or higher than. The mathematical expression with unequal sides is known as an inequality in mathematics. Inequality is referred to in mathematics when a relationship results in a non-equal comparison between two expressions or two numbers.

Given,

Compound inequalities has no solution is:

Variables should be moved to the equation's left side:

-8x + 7x > -3 + 9

Combining similar terms

-x < -3 + 9

Determine the total or difference:

-x < -6

Divide the inequality's two sides by the variable's coefficient:

x > -6

To learn more about inequality, visit:

https://brainly.com/question/15064057

#SPJ1

Three coffees and two muffins cost a total of 7 dollars. Two coffees and four muffins cost a total of 8 dollars. What is the individual price for a single coffee and a single muffin? ​

Answers

wAnswer:

e

Step-by-step explanation:

Please help me submit this

Answers

The range for the following sets is:

{9.12, 9.33, 9.18, 9.12. 9.24} = 0.21

{10.2, 3.8, 21.9, 33.4, 31.8, 17.2} = 29.6

{0.5, 0.65, 0.22, 0.11, 0.47, 0.55, 0.55, 0.71} = 0.54

{6, 2, 14, 11, 28, 10, 19, 15, 11} = 26

What is a range in a set of data?

The range is the difference between the highest value and the lowest value of a given set of data.

We have,

{9.12, 9.33, 9.18, 9.12. 9.24}

Range = 9.33 - 9.12

Range = 0.21

{10.2, 3.8, 21.9, 33.4, 31.8, 17.2}

Range = 33.4 - 3.8

Range = 29.6

{0.5, 0.65, 0.22, 0.11, 0.47, 0.55, 0.55, 0.71}

Range = 0.65 - 0.11

Range = 0.54

{6, 2, 14, 11, 28, 10, 19, 15, 11}

Range = 28 - 2

Range = 26

Thus,

The range of the given sets is as follows:

0.21, 29.6, 0.54, and 26.

Learn more about the range here:

https://brainly.com/question/15953457

#SPJ1

Write the equation of a circle with center (5, - 4) that intersects the point (5, - 2) .

Answers

The equation of the circle that has a center at (5, - 4) and intersects the point (5, - 2) will be (x - 5)² + (y + 4)² = 2².

What is a circle?

A circle is a geometrical figure which becomes by plotting a point around a fixed point by keeping a constant distance.

In our daily life, we always see circular objects for example our bike wheel.

The longest line which can be drawn inside the circle will be the diameter.

The equation of a circle with radius r and center (h,k) will be given as,

(x - h)² + (y - k)² = r²

As per the given circle,

(x - 5)² + (y + 4)² = r²

Since, it passes through (5,-2) so it must satisfy this point.

Put (5,-2)

(5 - 5)² + (-2 + 4)² = r²

r = 2

Thus, the equation of the circle will be (x - 5)² + (y + 4)² = 2².

Hence "The equation of the circle will be (x - 5)² + (y + 4)² = 2²".

To learn more about the circle,

https://brainly.com/question/266951

#SPJ1

please help me and be for reallll thankss ill give brainliest

Answers

If you add all you will get 50. Since your looking for C=8.

(Probability)

So 8/50=0.16

Round = 16. X

Brainly pls

A group of friends went to buffalo wild wings for dinner. There were nine people in your group. some of the group ordered chicken wings for $14.99 and the rest of the group ordered the chicken wrap for $8.49. If the total bill was $115.41, how many people ordered each dinner?

Answers

6 people ordered chicken wings and 3 people ordered chicken wrap.

What is an expression?

Mathematical expressions consist of at least two numbers or variables, at least one arithmetic operation, and a statement. It's possible to multiply, divide, add, or subtract with this mathematical operation. An expression's structure is as follows: Expression: (Math Operator, Number/Variable, Math Operator)

Given that there were 9 people in a group. Some of them ordered chicken wings and rest people ordered the chicken wrap.

The cost of chicken wings is $14.99.

The cost of chicken wrap is $8.49.

The total bill was $115.41.

Assume that, x number of people ordered chicken wings.

Then (9 - x) number of people ordered chicken wrap.

The cost of 1 chicken wings is $14.99

The cost of x chicken wings is $14.99x.

The cost of 1 chicken wrap is $8.49.

The cost of (9 - x) chicken wrap is $8.49(9 - x).

The total cost of x chicken wings and (9 - x) chicken wrap is $[14.99x + 8.49(9 - x)]

= $[14.99x + 76.41 - 8.49x]

= $[6.50x + 76.41]      [ Subtract like terms]

According to the question,

6.50x + 76.41 = 115.41

Subtract 76.41 from both sides:

6.50x = 39

x = 6

Putting x = 6 in the expression (9 - x)

9 - x = 9 - 6 = 3

To learn more about linear equation, click on below link:

https://brainly.com/question/21443645

#SPJ1

3. The length of each side of an equilateral triangle is (4x 2 7) cm.
Write an equation to determine the value of x that will produce a
triangle with a perimeter of 27 cm.

Answers

Answer:

Step-by-step explanation:

uhmm 4x +27 ?  4 times 27? what number is that supposed to be

let me try

4x + 27 + 4x + 27 + 4x + 27    (equilateral means all sides r the same length so 3 times cuz 3 angles in triangle)

12x + 81 = 27 (ok its prob not 4x +27 since x would be a negative number but just change 27 to the number it actually is)

12x = -54

x = -4.5

there enjoy it should be right if the numbers u gave me r correct

If two sides lengths are 36 inches and 45 inches, then find the potential third side lengths that could form a triangle. Group of answer choices

Answers

The potential third side lengths that could form a triangle lies in the range 9<x<81.

Given that, two sides lengths are 36 inches and 45 inches.

What is the triangle inequality theorem?

The triangle inequality theorem describes the relationship between the three sides of a triangle. According to this theorem, for any triangle, the sum of lengths of two sides is always greater than the third side.

Here, the potential third side lengths that could form a triangle.

So, the range of third side lengths(x) is 45-36<x<45+36

⇒ 9<x<81

Therefore, the potential third side lengths that could form a triangle lies in the range 9<x<81.

To learn more about the triangle inequality theorem visit:

https://brainly.com/question/1163433.

#SPJ1

Cheatem loans will lend you $400 with the understanding that you will repay them $500 in one month. What is the APR for this loan?

Answers

The APR for the loan is 300%

How to determine the APR for the loan?

APR stands for annual percentage rate. It is a standardized way of measuring the cost of borrowing money. It takes into account not only the interest rate on a loan but also any fees that are charged as part of the loan.

Given: Cheatem loans will lend you $400 and you will repay them $500 in one month

interest = 500-400 = $100

duration = 1 month

1 year = 12 months

APR = (interest/amount borrowed) / (duration/duration in a year)

APR = (100/400)  /  (1/12)

APR = 300%

Learn more about annual percentage rate on:

https://brainly.com/question/11686424

#SPJ1

What is 1/3 of a figure?

Answers

One third of a figure would be 1/3 of its total size or area.

The area of a form or planar lamina is referred to as the plane region or plane area, as opposed to surface area, which denotes the area of an open surface or the boundary of a three-dimensional object.Area can be thought of as the volume of material, at a particular thickness, required to create a model of the shape, or as the volume of paint required to paint the surface in a single coat. It is equivalent to the volume of a solid or the length of a curve in two dimensions (a three-dimensional concept).

1. A figure is a shape or an image that can be drawn or represented on a flat surface.

2. 1/3 of a figure would mean dividing the figure into three equal parts.

3. The resulting parts would be 1/3 of the size or area of the original figure.

Learn more about area here

https://brainly.com/question/13194650

#SPJ4

what is 4 Divided 1/3

Answers

Answer: 12

You're Welcome :D

Answer:

4 / 1 /3 = 4/3

Step-by-step explanation:

The distribution of heights in a population of women is approximately normal. sixteen percent of the women have heights less than 62 inches. about 97.5% of the women have heights less than 71 inches.Use the empirical rule to estimate the mean and standard deviation of the heights in this population.

Answers

The mean is 65 inches and the standard

standarddeviation is three (3) inches for the women have heights less than 62 inches. about 97.5% of the women have heights less than 71 inches.

In a set of girls, the distribution of heights is more or less normal. Women who're shorter than sixty two inches in top make up 16% of the population.

The empirical rule, much less than µ - o of 16% of the records is accurate.

P(x < mu sigma) =16%>mu - sigma = sixty two -> (l)Approximately 97.5% of the girls are beneathneath seventy one inches in top,P(x < mu sigma) =97.5%>mu + 2sigma = seventy one (||)By solving (l)& (ll) ;mu - sigma = 62u + 2sigma = 713sigma = 9sigma = 3From (1);mu - sigma = 62mu - 3 = 62mu = 65Hence, Mean = 65 inchesStandard deviation = 3 inches

To read more about Mean here:

http://brainly.com/question/23631778

#SPJ4

a population of cattle is increasing at a rate of 200 10t per year, where t is measured in years. by how much does the population increase between the 2nd and the 7th years?

Answers

The population increase between the 2nd and the 7th years as 440.

What is Total Change from a Rate of Change?

We may be able to utilise a function that expresses a rate of change to determine the total change in the quantity being described. This is so that we can take a rate of change and return to a function that defines the initial quantity. An indefinite integral will provide us with a function that depicts the whole quantity, while a definite integral will inform us of the total change between two places.

Total increase = ∫ab rate dt

= ∫₁³ 200+10t dt

= 200t +5t² |₁³

= [200(3) + 5(3)²] - [ 200(1) + 5(1)²]

= 600 + 45 - 200 -5

= 440

Hence, the population increase between the 2nd and the 7th years as 440.

To know more about rate of change check the below link:

https://brainly.com/question/8728504

#SPJ4

please help me with this, u dont even have to explain ur answer

Answers

Answer:

perpendicular bisector __D ,angle bisector__C

Step-by-step explanation:

1.becausein choise c the angle A is been bisected 2.am sorry but i can only answer the two questions as for the perpendicular bisector i guess it is self explanatory am sorry if i have not assisted

What is the velocity of a bus that travels 120 km in 3 hours?

Answers

The velocity of a bus that travels 120 kilometres in 3 hours is 40 kilometres per hour.

The velocity of bus will be calculated using the formula -

Velocity = change in position or displacement ÷ change in time

The change in position is 120 kilometres and the change in time is 3 hours, as both started from 0, the difference will be the number itself..

Keep the values in formula -

Velocity = 120/3

Performing division on Right Hand Side of the equation

Velocity = 40 kilometres per hour

Hence, the velocity is 40 kilometres per hour.

Learn more about velocity -

https://brainly.com/question/25749514

#SPJ4

Other Questions
price of two games in ratio a:b. If both prices decreased by 10, ratio = 3:5. If both prices increase by 5, ratio = 3:4. Work out ratio a:b. On a test that has a normal distribution, a score of 33 falls two standard deviations above the mean, and a score of 24 falls one standard deviation below the mean. Determine the mean of this test. what is the name for a predefined framework that can be used for controlling access, and is embedded into software and hardware? WILL GIVE 50 POINTS PLEASE HELPWhich gives the correct values for points A, B, C, and D?A number line going from negative 2 to positive 2 in increments of 1. There are 4 equal spaces between each number. Point A is at negative 2. Point B is 1 mark to the right of negative 2. Point C is 1 mark to the left negative 1. Point D is 1 mark to the left of 0.A = negative 2, B = negative 1 and three-fourths, C = negative 1 and one-fourth, D = negative one-fourthA = negative one-fourth, B = negative 1 and three-fourths, C = negative 1 and one-fourth, D = negative 2A = negative 2, B = negative three-fourths, C = negative one-fourth, D = one-fourthA = negative one-fourth, B = negative 1 and one-fourth, C = negative 1 and three-fourths, D = negative 2 What is the importance of being good and active Filipino citizens to our country? What is the answer to this please help asap Work out the value of x . The diagram is not drawn to scale. bruce, whose primary job is supervising a small production group, is not getting cooperation from all members on the cross-functional team he leads. in particular, selas, a senior marketing manager, seems to resist his direction and tries to influence team members to go in another direction. the source of conflict in this case may be multiple choice workplace incivility. ambiguous jurisdictions. inconsistent goals. status differences. How are the details about poseidon and athena's characters important to the development of the myths theme? cite evidence from the text in your answer. What will happen to the value of emf of the cell Zn Zn2+ 0.1 M cu2+ 0.1 m )/ Cu if the concentration of the electrolyte in the anode compartment is increased? rom a club of 12 people, in how many ways can a group of two members be selected to attend a conference? explain how economic development in the british north american colonies was impacted by the atlantic trade routes between 1650 and 1750. mg(oh)2, also known as milk of magnesia, is sometimes used as an antacid. determine the volume of stomach acid (2.90 m hcl) in milliliters that can be neutralized by a 25.0 g mg(oh)2 tablet. this patient was admitted for chemotherapy due to a primary hepatocellular carcinoma of the transplanted liver. what codes are assigned? What is the major message of the poem? bramble corp. recorded operating data for its shoe division for the year as follows: sales $14500000 contribution margin 580000 controllable fixed costs 160000 average total operating 1160000 assets what is the controllable margin for the year? 50% 25% $420000 $580000 a liter of cold water will warm faster in sunlight in a group of answer choices silver-colored pot. black-colored pot. red-colored pot. effective execution requires managers to build a foundation for execution within which three core processes found in any business? which of the following choices are characteristics of stock options? (check all that apply.) multiple select question. employees may choose not to exercise their options if the market value of the shares is below the strike price. if the market price is below the strike price, the employee may purchase the stock at fmv and sell it to the company for the option price. employees may exercise their options by paying the strike price to the employer anytime between the vesting and expiration dates. the employee will receive the stock on the vesting date without having to pay for it. employees may purchase the stock on the grant date, but can not sell their shares until the vesting date. an air freshener company created a marketing strategy after analyzing its various resources within the company. this is an example of a marketing strategy: